Paper B e-EQE 2021: The jury is still out

This year’s paper B was very difficult. The client's wishes and proposed amendments could scarcely be reconciled with the disclosure of the application as filed. Even among DP's experienced attorneys and tutors, this year’s paper led to dramatically different solutions.

Although it was promised that "Paper B will have the same syllabus and character as before", Paper B looked different in design from previous years. This year’s candidates were confronted, for the first time, with third-party observations, a computer-implemented invention (or not?), and client's claims with unusual (and incomplete) tracking of changes. In particular, the accumulation of these issues appears to have placed a very high burden on the candidates.

So far, we have been able to draw up several "solutions", each of which has its strengths and shortcomings - both in the light of meeting the wishes of the client, as well as good defensibility against the requirements of Art. 123(2), and with a clear narrative for the problem-solution approach.

For that reason, we will present below, for the first time, several proposals for amended claims, and our considerations thereof. 

Claim 1 could have been amended as follows:

1. A container (1) for organic refuse, comprising

a. an upper compartment (3) having a support (3a) for holding the refuse (7) populated with earthworms (8);

b. a lower compartment (4) for collecting excess moisture (6), wherein optionally the lower compartment (4) is removable;

c. drain holes (5) permitting moisture to drain from the upper to the lower compartment, or when the lower compartment (4) is removable directly into soil if the upper compartment (3) is placed directly on the soil; and

d. an upwardly opening covering lid (2) which fits over the container (1), wherein the container comprises a water spraying device (13) connected to a water container (14).


  • The client had removed "for organic refuse", but there only seems to be support for a container for "organic (kitchen) refuse"
  • The client had added "and/or housefly eggs"; while there is basis for (earth)worms and optional additives (cf. [022]), which may be  housefly eggs (cf. [003]), there seems to be no basis for "housefly eggs" alone as implied by the "or" language; the "housefly eggs" as additives can be put in a dependent claim
  • The client had deleted the "lower compartment" and the "drain holes", because, according to the client, "the lower compartment and the drain holes are both optional and are not essential to the container
    • deleting both the lower compartment and the drain holes, without adding a replacement feature, seems to certainly contravene Art. 123(2) as the invention clearly requires excess water to be drained. A possible solution is to replace both features by a more functional definition, e.g., “means for draining moisture from the refuse”, with possible support from [020] combined with [004], but ultimately we expected it not to be directly and unambiguously derivable from the application as filed, i.e., to contravene Art. 123(2) (and it would be very much against the established style of Paper B in which the wording of amendments are directly derivable from the wording of the application as filed - including for functional claim language)
    • the drain holes seem essential, because they not only are a means for controlling the moisture of the refuse, but they also prevent the earthworms from falling from the upper compartment into the lower compartment (and the soil) (cf. [017])
    • the lower compartment is trickier: both embodiments described and in the drawings have the lower compartment. But [018] says it may be removable; does this mean that it is non-essential and can be deleted? Indeed, we can argue that it was not explicitly explained as essential, is not indispensable for the invention (it can e.g. also drain on soil) and no compensating features are required for its removal - but is a container with only an "upper compartment" directly and unambiguously derivable from the application as filed as being an invention per se? 
    • The present amendment "wherein optionally the lower compartment (4) is removable" is our safe bet (and not completely in line with the client's wishes) - but we feel that a claim without the lower compartment and a convincing Art. 123(2) argumentation should get (more?) marks, too
  • The client had added "a water spraying device (13) for adjusting the moisture" and argued that this created novelty over D2 as "in D2 the water spraying device is exclusively used for cleaning" That argument does not hold, since the spraying device of D2 - attached to a tap - is of course also suitable for adjusting the moisture. [012] mentions a water spraying device (13) connected to a water container (14)
    • it is easy to see that the amendment "connected to a water container" creates novelty over the tap of D2. It also allows arguing a convincing PSA starting from D1 as CPA: a skilled person seeking to improve moisture control in the refuse, would not consider D2 as it is concerned with removing water, and it merely teaches to use a spraying device connected to a tap for cleaning the container
    • however, we can also understand that candidates struggled with adding the "water container" part to the claim, as this seems to be an external component to the container (see Fig. 2), and therefore seems to modify the claim in a "kit-of-parts". We also struggled with that, and with the fact that it results in a claim that is easy to circumvent.

An alternative solution could be:

1. A container (1) for organic refuse, comprising

a. an upper compartment (3) having a support (3a) for holding the refuse (7) populated with earthworms (8);

b. a removable lower compartment (4) for collecting excess moisture (6);

c. drain holes (5) permitting moisture to drain from the upper to the lower compartment, or directly into soil if the upper compartment (3) is placed directly on the soil; and

d. an upwardly opening covering lid (2) which fits over the container (1), wherein the container comprises a water spraying device (13)  for spraying water on the refuse


  • Novel over D1 (no spraying device at all) and over D2 (no removable lower compartment) and similar PSA story, with the addition that a removable lower compartment is undesirable w.r.t. larvae being present there
  • But: rather limited w.r.t. to the original claim, and also not fully in line with client's wishes.
  • In this solution, the connection to the water container is removed, but could also be included (see above for arguments; if you conclude that the water container as such is not indispensible for the function it serves, which is to supply water to the water spraying device, as it could be any type of water source/supply)

Further possible amendments (possibly as alternative solutions to limiting claim 1 to "connected to water container" and/or "removable") were:

  • a moisture detector (with a display)
    • In D1, but not in D2
    • present in all embodiments that use a water spraying device
    • seems required for the moisture data step of the calculation method
    • not suggested in any way by client, so rather arbitrary (not B style) amendment
  • adjustable size of the drain holes (cf. [017])
    • it contributes to the invention's effect of active control of the moisture
    • however, again arbitrary in view of the client's wishes

As far as the other (independent) claims are concerned:


The claim to a method for producing a fertilizer (original claim 4) was amended by the client to include

  • "and/or housefly eggs"; as for claim 1, not supported
  • "earthworms" instead of "worms"; this amendment (supported by [006]) is indeed required to overcome the Examiner's objection 
  • a backreference to the preceding container claims; that makes sense for restoring novelty over D1 and D2
  • specifying "by spraying water on said refuse" (NB: not tracked, and not explained by the client!) in step b


The following claim seems to be an acceptable amendment, also taking into account the removable lower compartment/soil option of amended claim 1:

X. A method for producing a fertilizer, the method comprising the following steps

a. providing a container (1) according to any of claims 1-xx, comprising organic refuse (7) and earthworms (8);

b. optionally, adjusting the moisture of the refuse by spraying water on the refuse and/or by draining moisture from the refuse;

c. composting the refuse with the earthworms for a time sufficient to convert the refuse into fertilizer, and,

d. optionally, separating earthworms from the fertilizer.

  • the step "and/or by draining moisture from the refuse" is likely not required because it can also be achieved automatically by the container. Novelty is already conferred by the use of the container of the invention itself


Finally, again  - in our view - two viable options for dealing with "mental act" claim 6:

9. A computer-implemented calculation method for optimizing processing organic refuse, comprising the steps of

a. defining a target value TV at time point tp > 0 for the amount of the earthworms (8) present in the refuse (7) at said time point tp;

b. receiving data relating to the moisture of refuse (7) and the amount of earthworms (8) at a plurality of time points tp;

c. determining if the value for the amount of earthworms (8) present in the refuse (7) at tp > 0 is equal to the defined target value TV for the amount of earthworms (8); and

d. recommending adjusting the amount of moisture added to the refuse (7) at said time point tp, if the value for the amount of earthworms (8) present in the refuse at tp > 0 is not equal to the defined target value TV for the amount of earthworms (8).

 

  • Basis in [024]: "One, more or all steps of the method can be implemented by a computer"
  • GL G-II 3.5.1: "Where a claim defining a method for performing mental acts as such is limited by specifying that the method is carried out by a computer, not only the use of a computer but also the steps carried out by the computer themselves may make a technical contribution if they are based on technical considerations and serve a technical purpose"
  • The main argument against this claim is that a large majority of EQE candidates is not familiar with computer-implemented inventions; on the other hand, any EQE candidiate is expected to be familiar with the main concepts of any technical field at a base level when sitting the exam, and you do not need to be more familiar CII than what the Guidelines and G 3/08 tell to be able to amend this claim as proposed above. In that sense, CII is not different from, e.g., second medical use.

 

However, based on the proposal of the client, the following claim should also be allowable:

X. A calculation method for optimizing processing organic refuse, comprising the steps of

a. providing a container according to any one of claims 1 to xx comprising organic refuse and earthworms,

ba. defining a target value TV at time point tp > 0 for the amount of the earthworms (8) present in the refuse (7) at said time point tp;

cb. receiving data relating to the moisture of refuse (7) and the amount of earthworms (8) at a plurality of time points tp;

dc. determining if the value for the amount of earthworms (8) present in the refuse (7) at tp > 0 is equal to the defined target value TV for the amount of earthworms (8); and

ed. recommending adjusting the amount of moisture added to the refuse (7) at said time point tp, if the value for the amount of earthworms (8) present in the refuse at tp > 0 is not equal to the defined target value TV for the amount of earthworms (8).

 

  • It should be specified that the container comprises organic refuse and earthworms as the container of claim 1 itself does not necessarily has them present
  • With the container in the claim, the claim can no longer be objected to under Art.52(2)&(3)
  • GL G-II 3.5.1: "A claimed method is not a method for performing mental acts as such if it requires the use of technical means (e.g. a computer, a measuring device, etc.) to carry out at least one of its steps or if it provides a physical entity as the resulting product"
  • With the container with the earthworms and the organic refuse in the claim, also the earthworms and the organic refuse are no longer abstract concepts but real things with technical character, so that also the further method steps are clearly technical and well-linked.
  • Main argument against this claim is that the "steps can be executed by a computer" information is not used.

With respect to the third party observations, we note that:
  • The announcement was published on the internet under www.opendayincotsford.com on 6 June 2018 does not disclose any container
  • Even though there were over 100 visitors who were impressed by the guided tour on 02 August 2018 at 11:45, where the public had access to the facilities and where the apparatus of claim 1 was located, the public could not see the internals of the container, not could they see the drain holes at the bottom of the container. There is no indication that the public could hold or open the containers to inspect them. As these claimed features were not visible, the open day on 02 Aug 2018 does not impact novelty of inventive step of the claim.
  • Further, there are no details given as to how the public could "learn how our earthworms and house flies produce fertilizer and feed", nor any evidence.

We did not want to delay posting a reasonable effort; however, because we feel we still may have overlooked something, we may post an updated answer in a few day's time. We will also provide at a later date (more of) our findings, including the dependent claims.

We look forward to your comments!
Comments are welcome in any official EPO language, not just English. So, comments in German and French are also very welcome!

Please do not post your comments anonymously - use a nickname (or your real name) instead - it makes responding more difficult and rather clumsy ("Dear Mr/Mrs/Ms Anonymous of 02-03-2021 22:23"), whereas using your real name or a nickname is more personal, more interesting and makes a more attractive conversation. You do not need to log in or make an account - it is OK to just put your (nick) name at the end of your post. Thanks!

The DeltaPatents Team


Note: you may also wish to check the comments posted to our first impressions blog, here.

Comments

  1. Hi,
    wherein optionally the lower compartment (4) is removable; = >> this is not limiting
    and then we again refer to non-limiting feature in the following feature.

    For the second option where you write "b. a removable lower compartment (4) for collecting excess moisture (6);" does this leave out the embodiments which comprise a non-removable container ?

    pfff- so confused

    What about detector and the display ? Is the original claim 1 basis for not adding it ?

    ReplyDelete
  2. Regarding the TPO, novelty of the device of (previous) claim 1 has been denied therein. In fact, it was stated that the device of (previous) claim 1 has been shown to the public. Since new claim 1 is novel over previous claim 1, the novelty objection raised in the TPO should become moot anyway, right?

    ReplyDelete
    Replies
    1. Whether moot or not, you will need to address it in your reply.

      Delete
    2. Sure, this is exactly what I have done. I Just wonder whether this line of argumentation is acceptable as well, as it has not been mentioned in the proposed solution above.

      Delete
  3. Thank you for your analysis!

    It seems to me that a claim to a "removable" lower compartment could be easily circumvented by a competitor providing a device with a lower compartment that is not removable. For that reason I removed the lower compartment but made the (size-adjustable) holes to be in the base of the upper compartment - this is to cover both embodiments and to not offend against 123(2).

    I didn't even notice that the client wanted changes to the optional steps in the second independent claim! If we sent out a response in real life that had not been proofread, that would be completely unacceptable, and rightly so. I would have expected the same standard from the EPO.

    ReplyDelete
  4. Why don't you have as one of the variants claim 1 based on the feature that drain wholes are for preventing the earthworms from falling (down). After all, it seems to be the only feature which you mention as essential, and drain holes are present in any of your claims anyway.

    ReplyDelete
    Replies
    1. I was wondering the same. I made this amendment to the drain holes too

      Delete
  5. Hello. Is the amendment to introduce the device into claim 4 directly and unambiguously disclosed? I mean, is the combination of the the method with that specific device directly and unambiguously derivable?

    ReplyDelete
    Replies
    1. See para [010] Brief description of the drawings:
      "Fig. 1 is a schematic drawing of the container comprising organic refuse and
      earthworms according to claim 1 which can be employed in a method for producing a
      fertilizer" Besides, the same reference signe are used... also in the calculation method

      Delete
    2. For the calculation method I agree, but for the other method no reference signs were used, and it was explicitly introduced as "another embodiment". The mention that the device can in general be used for producing a fertilizer does not mean directly and unambiguously that is or can be used in that specific method with those specific features

      Delete
    3. Reference signs are used in claim 4 in connection with the method for producing a fertilizer

      Delete
    4. Yes, but reference signs in the claims are not limiting. Is that really enough for direct and unambiguous disclosure in the originally filed application?

      Delete
    5. Thanks DP for your solution! I also wonder whether the use of same reference signs provides direct and unambiguous basis for introducing in claims 4 and 6 a back-reference to claim 1. In addition, at least for claim 6 this seems an unnecessary limitation of scope, since the computer-implemented calculation method without the reference to claim 1 seems anyway patentable (first hurdle of of G1/08) and the feature of recommending adjustments based on target values is new (including over D3) and provides a technical effect of allowing better control of moisture of organic refuse in a container (second hurdle of G1/08).

      Delete
  6. Have you considered the following alternative amendment for claim 1:
    "wherein the container comprises a water spraying device and the moisture detector".
    Reason for this:
    - it goes in the direction of the wishes of the client;
    - novelty & IS over D1 and D2
    - 123(2): the water spraying device and the moisture detector were disclosed together it seems to me that the moisture detector is essential for operating the water spraying device
    - the water spraying device and the moisture detector were disclosed together with the display and the water container, however one could argue that the skilled person could easily know that they are not essential because the last paragraph a sound alarm is given as an alternative to the display, whereas D2 uses a tap instead of a water container.
    I know this is a bit stretched but... A desperate patent calls for desperate measures!

    ReplyDelete
    Replies
    1. Isn't the moisture detector intrinsic in D2 due to mention of "reducing the moisture content of the manure by more than 50 percent" in par [001]?

      Alternatively, it may be obvious due to Examiner's objection to original claim 3.

      Delete
    2. For novelty, "reducing the moisture content of the manure by more than 50 percent" does not directly and unhambiguously imply that the container comprises the moisture detector.

      For inventive step, original claim 3 only has the moisture detecture and not the water spraying device. Obviousness over D2 is not an issue because its spraying device is used for cleaning therefore it would make not sense to put a moisture detector for deciding when to spary water. Anyhow D1 is the CPA and the spraying device from D2 cannot be combined in the light of the technical problem because they are used for cleaning.

      Delete
    3. I also have chosen water sprayer and moisture detector and deleted the drain holes, because in one of the first paragraphs the description says that moisture controle can be done by various methods, e.g. drain holes, evaporation etc. thus, drain holes are not essential to carry out the invention...nonetheless, since time was very short I did not finish the problem solution approach for any of the claims.

      Delete
  7. How did the DP people get on with timing?

    I have a very similar claim 1, and provided extensive basis arguments for each claim, but didn't manage to get to inventive step other than for half of claim 1.

    In my view having three independent claims, each of which could be amended differently, three prior art documents and TPO made this paper a lot more time consuming than previous paper Bs. This is compounded with the fact most of the paper is not printable or highlightable, and amendments with square brackets and formatting issues take a lot longer to make.

    I would be pretty disappointed if I failed having got the right amendment, purely because the paper was a lot longer than previous years. I'm kicking myself to an extent because inventive step is the easiest place to hoover up marks, but clearly it cannot be done until the rest of the paper is completed...

    ReplyDelete
  8. Hi DP team, i posted this on the other blog. Please can you consider and let me know what you think.

    Concerning the paragraph “The size of the drain holes may be adjustable to allow active control of the moisture”, this appears to me to be unclear as there is no disclosure in the application as to how the size of holes can be adjusted. An EPO examiner would always request that structural limitations be introduced to quantify this term.

    With respect to the following passage “The lower compartment (4) may be removable, allowing the upper compartment (3) to be placed directly on soil, so that the excess moisture drains into the soil”, I am not even sure this is compatible with the containers in the Figures. Figure 2 clearly shows the upper and lower edges of the lower compartment. Underneath the lower compartment is the container base. Thus, if the lower compartment were removed would the upper compartment not simply rest on the base of the container. There is no disclosure in the spec to indicate that the lower compartment forms the base of the container. Also the base of the container appears to be solid (well it is a container). Thus, how can the upper container be located directly on the soil.
    So to clarify it appears that there is no basis for removing the lower compartment from claim 1 when including features from the embodiment disclosed in Figure 2 (i.e. the spray).

    ReplyDelete
  9. I do not agree with the statement “ deleting both the lower compartment and the drain holes, without adding a replacement feature, seems to certainly contravene Art. 123(2) as the invention clearly requires excess water to be drained.”

    In #3 or #4 there are alternative to reduce excess moisture instead of drain holes. When evaporation is used, you do not need drain holes. Opening lid or some other way can help evaporation and do not require modification of the container

    ReplyDelete
    Replies
    1. Hi, I agree with you, I also considered that it was allowable.

      In addition to what you pointed out, there was paragraph 20 stating that "The moisture of the refuse can be adjusted by spraying water on the refuse and/or by draining moisture from the refuse. This indicates that even removing water (by drain holes or the other possible means) is not necessary for the invention, as adjusting the moisture can also be done exclusively by spraying.
      See also paragraph 5 discussing conditions wherein only having spraying means would make sense: "If the refuse is too dry the worms will not survive. In most cases the organic refuse is moist enough to allow worms to compost the refuse. When the outside temperature is high and humidity is low, it is, however, necessary to adjust the moisture, e.g. by spraying water on the refuse."

      Delete
    2. I second that

      Delete
    3. I agree with you. In addition, in the client letter there is a strong push to remove the lower part and the drain holes, even suggesting what is obvious to a skilled person. Why should they have entered this clue? I don't want to believe it's pure sadism.

      Delete
  10. Thank you for this model solution!

    Do you think that the EPO wanted us to do a second problem solution approach for the "mental act" claim 6 or simply refer to independent claim 1? Depending on which of the two routes for "mental act" claim 6 is chosen, the problem solution approach should look rather different. Also, will the requirements of unity be met, i.e. is there a single inventive concept and a common special technical feature for the "mental act" claim with the remaining independent claims?

    With regard to the TPO and in addition to the TPO, it further seems that there is no indication that the client is cycle farm (where the tour was held). So there is no reason to believe that the container the public saw is the same one that the client developed.

    ReplyDelete
    Replies
    1. I was indeed concerned about unity. What can be the technical problem linking the two method claims? where are the common special technical features in the two method steps? I ended up deleting claim 6 and suggested a divisional.

      Delete
  11. Also, I agree with above that there is no basis for amending claim 4 to refer to claim 1. The passage referring to container being used in the methods is only in relation to Figure 1 and not figure 2. So we appear to combining embodiments. Can you consider DP please

    ReplyDelete
    Replies
    1. I also agree that there is no basis for adding the container into claim 4. The spec mentions a method that can be used with the container of claim 1, but if claim 1 has been amended to include the water sprayer etc, and there is no mention of the method being used with the container of the embodiment in figure 2, then how can there be basis for making it dependent on the new container claims?

      I was relying on the step of adjusting the moisture in the refuse by spraying water onto the refuse not being optional to overcome D2. D2 only describes the water spray cleaning device and does not disclose spraying water onto the refuse.

      I did novelty for the independent method claim but not inventive step, as I ran out of time

      Delete
    2. See para [010] Brief description of the drawings:
      "Fig. 1 is a schematic drawing of the container comprising organic refuse and
      earthworms according to claim 1 which can be employed in a method for producing a
      fertilizer" Besides, the same reference signe are used... also in the calculation method

      Delete
    3. Yes, but if you've put the water sprayer in amended claim 1, that was not originally in claim 1 (and no reference numerals for this feature in the method claim)

      Delete
    4. Why would we even want to refer claim 4 back to any of the previous claims? Isn't it already new and inventive without that because none of D1 to D3 suggests a moisture adjustment for earthworms by spraying water on refuse?

      Or did I miss sth?

      Delete
    5. In the exam report, there was no novelty objection over D2 in the first place for claim 4. Only D1 and D3, neither of which disclose spraying water on the refuse (required after deleting 'optional'), so I believed that no reference to the container of claim 1-X was needed for novelty, and was not disclosed in the application as filed anyway - [010] only referred to the method employing the container of (original) claim 1.

      Delete
  12. In para 003, it reads: "According to the prior art, composting of the refuse may be enhanced by
    adding additives like organic nitrogen sources, bacteria, fungi, housefly eggs or worms"... due to the term "or", it cannot be said that there is a basis for housefly eggs and (earth)worms, right?

    ReplyDelete
    Replies
    1. That is about prior art.

      Delete
    2. But without this text tassage, it is not possible to arrive at the embodement covering housefly eggs and earthworms, as suggested as a dependent claim?!

      Delete
  13. Thank you for your analysis, it was indeed a devastating paper. We spent alot of time trying to figure out a way out and therefore had no time for providing solid arguments for the IS. Please find below my thoughts and feel free to challenge. I did not pay much attention to formalities, but I hope the amendments are clear.

    1.A container (1) for organic refuse, comprising;
    a.an upper compartment (3) having a support (3a) for holding the refuse (7) populated with earthworms (8);and
    [b.a lower compartment (4) for collecting excess moisture (6);]
    b. [C].means for [drain holes] permitting moisture to drain from the upper compartment [ to the lower compartment, ] wherein the means is one of drain holes or moist absorber material (I do not know if evaporation could be added - sounds weird)
    c.[d].an upwardly opening covering lid (2) which fits over the container (1)
    wherein the container comprises a water spraying device (13) connected to a water container (14) and a moisture detector (11) with a display (12), for adjusting the moisture.

    (I have deleted lower compartment and argued that being removable shows that it is not essential for the OTP invention serves to solve. In addition, it is only function is to collect excess moisture which we do not need to be collected for any reason. I have used a functional wording "means" instead of drain holes and then kept drain holes and the moist absorbent as limiting alternatives in order not to violate A123(2). I thought moisture detector (11) with a display (12) required for achieving the effect of actively adjusting the moisture, also another reason is that IS would be too thin over D1+D2 without them. I wanted to leave out display but there was no basis to do so, only in last parag. of the desc. an alarm is mentioned but for a different embodiment. )

    I fully agree with your claim 4. I have done exactly the same.

    For claim 6, the last claim about CII, (I am not a CII specialist) I do not see any basis for your second option for this claim. Because the paragraph which mentions this calculation method starts as “another embodiment”. There is no connection, unfortunately. Furthermore, in the following parag. the language is in a way like “a container” “a detector” as if it was not the other embodiment of the invention. So totally got confused. Left out referral to the other claims just written that:

    6. A computer implemented method for optimizing processing organic refuse, comprising the steps of
    a. defining a target value TV at time point tp > 0 for the amount of the earthworms (8) present in the refuse (7) at said time point tp;
    b. receiving data relating to the moisture of refuse (7) and the amount of earthworms (8) at a plurality of time points tp;
    c. determining if the value for the amount of earthworms (8) present in the refuse (7) at tp > 0 is equal to the defined target value TV for the amount of earthworms (8); and
    d.recommending adjusting the amount of moisture added to the refuse (7) at said time point tp, if the value for the amount of earthworms (8) present in the refuse at tp > 0 is not equal to the defined target value TV for the amount of earthworms (8).

    Now I also think that, last parag. could be a basis for adding some more technical character to the claim… by inserting detector, infrared camera, monitor or alarm to the relevant steps in a proper way. But there was no time to tune this claim.

    Time was really an issue... TPO was not a big thing to deal with but it was the first thing triggered the excess stress at the beginning, then with the other demand and problems of the paper, everything turned into a storm unmanageable in 3,5 hours...

    ReplyDelete
    Replies
    1. I agree, If lower compartment removable and upper compartment can be directly on the soil, then can claim just upper compartment ik independent claim and have a dependent claim to lower compartment and a thereon dependent claim with a removable lower compartment.

      C.

      Delete
    2. @Begum: I agree with almost everything and have answered much in the same way, but what about unity of invention? In what way does CII claim 6 is unitary with the device and method of claims 1 and 4? Furthermore, aren't there essential technical features missing in claim 6 for carrying out the method? You are right about not finding the connection between the CII method and the device in the description. I was also struggling to argue for a basis to introduce the container of claim 1 into claim 6.

      Delete
  14. I did broadly as the client wished, adding "the container comprises a water spraying device (13) for spraying water on the refuse" to claim 1 and removing the two steps (argued this was ok for a123), but didn't make the housefly amendment. That addition gave a nice inventive step argument over D1+D2. I was then an idiot for not making claim 1 clearly novel over d2.i think i should have included the moisture detector, this seems, in retrospect, in line with the client wishes because it was retained in claim 2. This also seems synergistic with the addition to claim 1 (albeit that it's not a distinguishing feature over D2 And so not synergistic in the inventive step sense)
    By the way, I argued the removal of the holes etc from claim 1 as allowable because the description gave various ways by which water could be removed, including evaporation and merely opening the lid in addition to the holes.

    ReplyDelete
  15. Lower compartment in D2 should somehow be removable to get out the larvae right?

    ReplyDelete
    Replies
    1. But D2 does not disclose it. Thus, this is only speculation. There are also many other ways to get larvae out, e.g. via opening in the lower compartment etc.

      Delete
  16. Thanks a lot for your effort, DP. I introduced claim 2 into claim 1 as differentiating feature. The Office Action stated that claim 2 did not comprise inventive features, but when trying to get to the subject-matter of claim 2 by combining the prior art documents, I failed since the features of claim 2 were not disclosed in any of documents D1-D3. I would be really grateful if you could give me your opinion on this approach. I'm sure it is not the "best" solution, but I still believe the claim is novel and inventive because the prior art does not disclose the covering with the strings attached to the lid. I would really like to hear/read another opinion on this :)

    ReplyDelete
  17. Thank you for taking the time and effort in drafting your solutions!

    Did you consider in claim 1 to substitute the drain holes with the water absorbent material? The client says "If possible, however, we do not want to limit our invention to water absorbent material, because it is less convenient for the operator." However, the proposed claim is not possible due to lack of novelty over D2.

    Furthermore, he says that the proposed claims satisify their buiseness needs. Your suggested water container is not in the proposed claims. Shouldnt we conclude from this that a limitation to the water container does not satisfy the buisiness needs of the client?

    ReplyDelete
    Replies
    1. Further to my comment above, keeping the lower compartment in claim clearly goes against the wishes of the client: "the lower compartment and the drain holes are both optional and are not essential to the container. They can therefore be omitted".

      How do feel about the following argument: the drawer (16) of D1 is a removable lower compartment. Hence, your alternative solution lacks novelty over D1?

      Delete
    2. Excuse me, lack inventive step over D2+D1 (i.e. add the removable drawer of D1 to D2)?

      Delete
    3. Wow Jan you are entirely right. I checked and the removable drawer in paragraph 5 of D1 is pretty damning.

      Delete
  18. The feature "connected to a water container" in claim 1 is non-limiting due to F-IV, 4.14 "Definition by reference to (use with) another entity".

    Most likely, it cannot provide novelty over D2, unless rewritten according to Guidelines "Only if the claim is directed Without Any Doubt to a combination of the first and second entities, the features of the other entity are limiting for the subject-matter of the claim. In the above example, the claim should be written as an "engine with a cylinder head" or an "engine comprising a cylinder head" for the features of the engine to be considered as limiting the subject-matter of the claim".

    ReplyDelete
    Replies
    1. I donot agree that the DP proposal would be an unclear reference to an external entity. It simply claims the combination. See F-IV, 4.14.1:

      F-IV, 4.14.1 14.1 Clarity objections
      Once it has been established if a claim is directed to either one entity or to a combination of entities, the wording of the claim must be adapted appropriately to reflect it; otherwise the claim is objected to under Art. 84.

      For example, in the case of a claim directed to a single entity, the first entity is "connectable" to the second entity; in the case of a claim directed to a combination of entities the first entity is "connected" to the second entity.

      C.

      Delete
    2. DP's claim 1 is not aimed at a combination of two entities because its preamble is aimed at one entity (the container).

      Put it simply: if DP wants container (1) to comprise a water container, then claim has to be amended to include such an expression, due to F-IV, 4.14.

      If DP does Not want container (1) to comprise a water container, then the word "connectable" instead of "connected" should be used.

      It is such a standard clarity objection that frankly there is nothing to discuss. They know about the problem, this is why they mentioned the "kit-of-parts".

      Delete
    3. As far as I can recall, the word "connectable" was not in the spec anywhere, only "connected". So you wouldn't have basis for "connectable". (I dealt with exactly this situation in real life and it wasn't allowed, no matter how much we argued.)

      Delete
    4. On second thought, that real life case might have had one device disclosed as connected to another all the time, so it might be a different situation compared to this paper where the device is also disclosed as not connected. I'm too tired to go and check.

      Delete
    5. V, "connectable" is not interesting in any case because it would not make claim 1 novel over D2.

      IMHO, to me it seems that DP wants the container (1) to be limited by the feature "connected to a water container" in a way as if such a container comprised the water container. However, imho, this is against the rule of interpretation in F-IV, 4.14 "Only if the claim is directed Without Any Doubt to a combination of the first and second entities, the features of the other entity are limiting for the subject-matter of the claim.".

      Overall, I do not understand, does DP thinks that the container (1) in their claim 1 comprises a water container, or not? If yes, this feature must be made explicit in a claim, due to F-IV, 4.14. And basis would need to be found. Fig. 2 does not seem to provide it.

      Delete
    6. I'm confused as well. In addition, I don't think including the water container is a good idea, see my comment later in the thread.

      Delete
  19. I'm really looking forward to the Examiners' report for this one. What a mess

    ReplyDelete
  20. In your first solution, you say: it is easy to see that the amendment "connected to a water container" creates novelty over the tap of D2.

    I'm sorry, but where do you think the water comes from before it reaches a water tap? Implicetly, there is always some water container containing the water before it reaches the tap. Even the water line connected to the tap contains water, and is therefore a water container.

    And consider the client: he now has a claim limited to a water spraying device connected to a water container. His application does not give any technical, surpising or advantageous effect of using a water container. A competitor connecting his device to a water tap would not infringe!

    ReplyDelete
    Replies
    1. keep in mind that most states have a doctrine of equivalence.

      Delete
    2. That is correct, in fact all EPC contracting states should take into account equivalents (Art. 69 and the Protocol).

      However, I would surely hope that during exam B, we should not take into account how a national court of a contracting state would possibly take into account equivalents!

      And there is also the Gilette-defense: if the patentee would argue that a water tap is equivalent (same function, way and result) to the water container, than the claim would surely lack novelty or at least an inventive step over D2 disclosing such a perfectly equivalent water tap!

      Delete
    3. This was my problem too,I thought the tap is implicitly connected to a water container. I added a feature along the lines of the drain holes prevented earthworms from passing to make it novel over D2 (and i didn't include the lower compartment in claim 1). But what a mess. Even if in the end many different solutions are accepted, it was so confusing to consider all the options, none of which seemed the right one,and so different from the previous B papers,I took far too long to decide on a way to go before starting my argumentation. Also wiseflow is really inconvenient for paper B without being allowed to print more of the paper.

      Mich

      Delete
    4. I agree 100%. Simply a tab without a water container anywhere would not deliver any water at all. Therefore any functioning cleaning device that is connected to a tab implicitly must bei connected to a water container.

      One must stay realistic when answering the exam questions. No real world examiner would accept such an immensely weak argument. I would be even ashamed to explain this to my client "I believe you'll be granted a patent because a water container with a line that can be connected to your spraying device is something completely different to a functioning water tab!?!"

      Delete
  21. A further point on the third party observations is that the tour schedule provides a programme for Sunday 2 August, but the newspaper refers to a tour on Monday 3 August. There is therefore no evidence that the scheduled tour on the Sunday even took place, as the tour on Monday may have had a different programme.

    ReplyDelete
    Replies
    1. True. But both dates would be prior art so I also did not mention this in my answer. It is anyhow the tours itself, and not what the newspaper writes, that is the possibly relevant prior art. Or?

      C.

      Delete
    2. Interesting point! The wording: "[005] According to the local newspaper “Cotsford Herald” (www.cotald.com) on Monday 3rd August 2018 the open day attracted approximately 100 visitors" is super ambiguous.

      Did the local newspaper report it on Monday 3rd August; or
      did on Monday 3rd August, the open day attracted approx 100 visitors?

      I noticed this to during the exam, and decided to go for the first option.

      The examination committee should have added a comma:
      [005] According to the local newspaper “Cotsford Herald” (www.cotald.com) on
      Monday 3rd August 2018, the open day attracted approximately 100 visitors
      [005] According to the local newspaper “Cotsford Herald” (www.cotald.com), on Monday 3rd August 2018 the open day attracted approximately 100 visitors

      Delete
    3. The French version has a comma after the date so I assume that is what was intended. I agree in English it is unclear - another error in the paper...

      Delete
  22. Another question/comment if you don't mind:

    You state:
    the drain holes seem essential, because they not only are a means for controlling the moisture of the refuse, but they also prevent the earthworms from falling from the upper compartment into the lower compartment (and the soil) (cf. [017])

    Wouldn't removing the drain holes also prevent earthworms from falling through the drain holes? I think [017] discusses that the size of the drain holes (if used) are large enough to allow draining of water, but small enough to prevent falling of the earthworms.

    If you remove drain holes, you just end up with a solid surface through which moisture nor earthworm can fall.

    ReplyDelete
    Replies
    1. I agree. Also one of the earlier pars. of the application stated that the skilled person knows there are other ways of draining the water. Nevertheless, I included the drain holes for patentability

      Delete
  23. Re "wherein the container comprises a water spraying device (13) connected to a water container (14)": such a claim would mean that anyone selling a container with a water spraying device that is not connected to a water container (which is surely exactly what you would be selling!) would not infringe the claim. The only people who would infringe are the end users and since they are also potential customers of the client surely the client would not want to go after them. So, a claim for a device connected to a water container seems commercially rather pointless to me.

    ReplyDelete
    Replies
    1. indirect infringement still applies - in the UK at least, the sellers would be providing means for putting the invention into effect so you could go after them too.

      Delete
    2. You don't want to have to rely on indirect infringement if you don't have to, though. Not least because then, in the UK, the double territoriality requirement kicks in.

      Delete
    3. I agree with your analysis, V.

      Double territoriality also applies in NL. Furthermore, I don't think the EPC mentions indirect infrigement at all, so I wouldn't expect this to be relevant to the EQE.

      Additionally, I do not see any advantage in using a water container over the water tap in the application. The water container is not disclosed to be portable (is it for containing 1 Litre, 100 Litres?), nor is there any indication that there are means present for controlling the flow of water between the container and the spraying device. The water tap at least gives you some control over the flow of water.

      Delete
  24. I dealt already quite a lot with CII during my training, since many of our clients are active in this field. Hence, I have seen many such Office Actions. I do not believe that the suggested claims 6 would bei allowed at all due to lack of inventive step. The examiner would simply deny the technicity of every step and only read "define a value -> receive data (twice) -> calculate another value based on the received data and the defined value". If the data is number of earthworms or anything else, is technically irrelevant. Basically, the examiner would say the programming of an algorithm to output a recommend moisture adjustment value is part of the ordinary daily tasks of a programmer.

    Further technical features like the infrared counting system etc. seem to be necessary.

    ReplyDelete
    Replies
    1. I agree. I added a step of displaying the recommendation on a screen or alerting an operator for the same reason: they can take action based on the information produced by the algorithm, which should count as a "further technical effect".

      Delete
    2. Agree with both of you. I included "computer-implemented", "moisture detector", "means for counting worms" (based on last par.), and "screen/alarm" for these reasons

      Delete
    3. I agree. The proposed claim passes the "per se" bar, and is thus an "invention", but the sole technical feature (i.e. the computer) is not sufficient to be inventive...

      Delete
    4. I also agree with that. Computer implemented would help to pass the first hurdle but merely playing with abstract data is not enough for IS. A diagnostic from actual measurement would.

      What I did was to indicate the infrared system was used for counting.

      By using the infrared device in the method, you pass the first hurdle. I also argued that it was easy to do the counting and avoided the need to take a sample for manual counting which is how it was done in D3 I think. I thought about the moisture measurement device instead of the infrared but I think this was written somewhere that this device was well-known.

      So I didn't even specify the method as computer implemented. Having already passed the first hurdle, I argued that the rest of the mental act steps have a technical contribution in the context of the claim in that they provide a recommendation helping the obtain the desired number of worms measured by the infrared.

      Now I know that my claim, not referring to claims 1-X, is non unitary with the container and the other method claim using the said container but I could not find a basis to do so (I could however find a basis for including the device claims 1-X in the other method claim).

      Delete
    5. I've been wondering about unity too. I referred to claims 1-X in the last claim but I didn't have time to look for any basis.

      Delete
    6. Yes, if it was that easy to get a "software patent" from the EPO by just changing the term "calculation method" to computer "implemented calculation method" I would ask myself why there are not much more of them?

      Delete
    7. In my opinion, the CI claims mist comprise the additional means as the steps are not performed by the data processing means. The means of paper B are comparable to the detector in the Guidelines. https://www.epo.org/law-practice/legal-texts/html/guidelines/e/f_iv_3_9_2.htm

      Delete
  25. I would argue that the fact the lower compatment can be removed and moisture can drain into the soil is proof that the lower compartment is not essential, can be detached without chances to other feature and, most importantly, discloses a working embodiment (container with disassembled lower compartment) that doesn't have the lower compartment.
    In that embodiment, the upper compartment is disclosed at most as "connactable" to a lower compartment, however even this feature is clearly not essential.

    ReplyDelete
  26. Thanks for working out the solutions so quickly,
    I do not think the limitation to the water container in claim 1 withstands an inventive step attack using D2 as a starting point. The problem to be solved would simply be to provide an alternative water supply for the water tap and the use of a container seems to be an obvious solution thereto. Candidates are well aware of the fact that any document could be used as CPA and in this case D2 is the more promising springboard because it already has a built-in water spraying device. Claim 1 is not defining how the water spraying device is to be used, i.e. it does not matter whether it is used to add moisture to the organic refuse or to clean.
    In my opinion, the only meaningful, but strong limitation to claim 1 is to specify that the drain holes are draining the excess moisture directly into the ground. In D2 this is not possible since the lower compartment has a closed floor.

    ReplyDelete
  27. If the mental act claim is not amended to refer to spraying, it would violate EPC as non-unitary with two other independent claims.

    At the same time, according to IPREE, Rule 24(3) the response shall be in the form of a letter to the EPO accompanied by the claims supplied by the client, amended as appropriate to meet the requirements of the EPC.

    ReplyDelete
  28. In D2: "The container (1) has an external light
    source (20) and a spray cleaning device (13) connected to a water tap (21)." That is, according to this wording, the light source clerly forms part of the container... so does the tap

    ReplyDelete
    Replies
    1. The tap in D2 is not necessarily comprised by the container, nor is the water container necessarily comprised by the container in claim 1 as proposed by DP.
      A careful claim interpretation always matters.
      A proper reading of claim 1 teaches only that the container includes a water spraying device. In addition thereto, claim 1 is further limited by the condition that the water spraying device is connected to a water container, i.e. there must be some sort of connection between them. This should be construed as a limitation which specifies the manner in which the container has to be installed, i.e. by connecting it to a water container.
      Beware that "connected to" is very general and comprises both direct and indirect connections; for instance a water container that is connected to the spraying device via a water tap! It also encompasses both releasable connections and permanent-type connections.
      The following examples give some guidance: your computer connected to the internet does not mean that your computer contains the whole internet, a power plant connected to the power grid does not mean that the power grid is a part of the power plant, a household appliance with a plug when connected to a socket does not mean the socket belongs to the appliance, etc.
      The result of this: the scope of protection conferred by claim 1 is illusory; any competitor will just sell the composting container and the water container as separate parts or as a kit of parts and your client cannot do anything about it. He will only be able to sue the end consumer for infringement, who actually establishes the connection when installing the container.
      Moreover, consider the fact that both the container as such is known from D2 as is a water container by itself. Would you consider it to be an inventive act if someone connects a water container to the container of D2 instead of connecting the tap? The anser is obviously NO and no patent should be granted on the basis of claim 1.

      Delete
    2. "The container (1) has an external light
      source (20)"... you agree that the container comprises the external light source? Even if it is depicted above the container?

      Delete
    3. Yes, in view of my above comments I would indeed affirm that the container in D2 comprises a light source. It clearly says container (1) has an external light source and for myself, I would equate having with including. This means that the container of D2, by definition, is a container which always has an external light source. The light source is depicted as a light bulb above the container - kind of freely floating in space - so the figure of D2 is a bit misleading in that it suggests a separate entity that is detached from the container. A better drawing could have been helpful here, i.e. kind of a light fixture that is permanently coupled to the container.

      Delete
  29. That's what I don't like about this paper. Too many plausible options that can be defended, but also attacked. Of course, if marks will be awarded for various well-argued solutions, this does not have to lead to bad results.

    I didn't believe that a reservoir was inventive (if even novel) over a tap (why not use a reservoir instead of a tap for cleaning), so I went for the moisture detector. I struggled with the monitor, which I really didn't want in the claim. However, I failed paper B before because I went too broad, so I'd probably keep it in to be on the safe side here.

    I was tempted to add the housefly eggs and introduce "means for removing excess moisture" based on [003] and [004]. The client really seems to hint she wanted both. But taking features from the background art section is at least problematic (https://www.epo.org/law-practice/legal-texts/html/caselaw/2019/e/clr_ii_e_1_11_3.htm), so I decided against it.

    I felt very unhappy for going against the wishes of the client on so many points and very happy that I don't need to sit the exam myself anymore. I hope the exam committee will take some freedom to give a substantial amount of marks for well-argued 'inferior' solutions.

    ReplyDelete
    Replies
    1. This was my problem (or one of my problems) during the exam too: I read the client's letter and thought well, they want this but I can't do that, nor that, nor that, nor...
      I added "and optionally housefly eggs", because and/or would have meant it could have been only housefly eggs, without earthworms, and there was no basis for that. I wasn't aware of the case law you quoted, though. It seems like even this wasn't a good idea, then.

      Delete
  30. If experienced tutors are not fully sure of what the right direction/amendments are - what hope does 2021 candidates have. The EQEs 2021 have been shambolic. Will any of it matter, I doubt it, the EPO examining committee do not care about candidates but would rather use them as Guibea Pigs for the fun of it so it seems. Epi need to support us this year during discussions with the EPO.

    ReplyDelete
    Replies
    1. We have been discussing for a couple of days... how shall one be sure after 3.5 hrs?!

      Delete
  31. What was the mock 2 paper for... when the real one is such different?!

    ReplyDelete
  32. Dear Joeri,

    You say: "Too many plausible options that can be defended, but also attacked."

    I'd say: "No plausible option that can be defended like you would normally be able to defend your amendments in a B paper, in terms of inventive step and commercial relevance for the client".

    I am curious how you would have defended inventive step of adding a moisture detector which is said to be standard, and the effect of which was already discussed in D1?

    I also considered your option of adding "means for removing excess moisture" during the exam, but I found this feature to already be disclosed in D1 by the drain holes, and possibly also in D2 since the larvae result in reduction of the moisture content. That why I went for the moisture absorbing material, which I feel at least was not known from D1, D2 or D3.

    ReplyDelete
    Replies
    1. First a disclaimer. I haven't followed paper B for more than a decade, so my decisions may be more based on real life practice than on what is expected in the exams.

      For the detector. The detector itself may be common, but not its combination with spraying means in a device like this. D1 misses the sprayer and is only concerned about removing moisture, not about adding. D2 misses the detector and a sprayer for cleaning does not need a detector. Also D2 is only concerned about reducing moisture content.

      In general, an inventive combination of features A+B, where D1 has A and D2 has B is exactly what I would expect in an exam.

      Delete
    2. its as easy as this: D1 is closest prior art disclosing a moisture detector. Thus, you have to argue inventive step of the water sprying device. Such device is presented in D2, but for a total different purpose.

      With respect to D2, there is no moisture detector present, because it is not needed for a method in which water should be reduced to arrive at a dry refuse. What would be the intention to include a moisture detector in this case?

      Delete
  33. I also think that claim 6, when amended into a CII, is not inventive over D3.
    Document D3 does not teach a CII, but clearly hints to a software implementation using the information (above all the information in the table of D3). This is a strong incentive for the skilled person to actually implement a CII. The tabulated data used in D3, when used in the software, implies receiving data on the earthworm population and moisture level at different points in time (e.g. after 30, 60 days, etc.). The row relating to the organic kitchen refuse would be used to define target values at time tp>0, e.g. 16 worms/per 1000g of refuse/20 earthworms initially loaded at day 90. Par. 5 of D3 teaches optimizing a healthy environment for the earthworms and that this environment is determined mainly by the moisture content and not so much influenced by the substrate type. Hence, the software would be easily adapted by the skilled person to recommend adjusting the moisture level if a user input for an amount of earthworms in his container at day 90 deviates from the target value of 16 worms/per 1000g of refuse/20 earthworms initially loaded, which serves as a reference for the software (it is natural to define these ratios or worm densities; the client's invention also relies on studying on worms counts in a limited-volume sample, and it is evident that the absolute worm population is proportional to the initial population, unless they all die).
    Would you agree?

    ReplyDelete
    Replies
    1. Totally agree. Hence why I added the infrared counting which was not mentioned anywhere else to aim for IS.

      Delete
  34. Also, I am not sure about D2 not disclosing a removable lower compartment. It does not say it explicitly, but does so implicitly in my opinion. How else does it make technical sense in D2 if the migrated larvae cannot be collected from the lower compartment in the end? And how does the cleaning make sense if all the dirty water would accumulate at the solid floor?
    I was confused about the terminology used in D2, e.g. is the mesh tray fitted into the lower compartment different from the mesh floor? Does the fitting mean that the mesh tray can be removed? If the mesh tray is removable and is the same as the mesh floor of the upper compartment, then this would imply that the lower compartment can be separated from the upper compartment...

    ReplyDelete
  35. There are so many justified objections in the above comments that I even doubt the suggested solutions would pass.

    ReplyDelete
  36. I believe the suggestion by client to remove display had no basis. In paragraphs discussing Fig 2 there was no disclosure that it was optional bar paragraph [024] but this was related to the method. Anyone else leave it in?

    ReplyDelete
  37. Is it necessary to amend claim 4 to be dependent on the previous claims to restore novelty over D2 if the communication only stated that claim 4 lacks novelty over D1 and D3? I just added the spraying to add moisture step because that restored novelty over D1 and D3.

    ReplyDelete
    Replies
    1. I would also think so. You usually do not have to respond to potential issues if they have not been raised by the examiner in the EQE. Otherwise, I could also start correcting spelling or typing mistakes of the application in the exam and expect marks for it.

      Delete
    2. Thanks for replying. I’m relieved to hear you agree. Panicked a bit as lack of novelty means 0 points..

      Delete
    3. including the spraying in cleim 4 also renders the claim inventive, because the intention of D2 is to dry the refuse. so there is no need in the method of D2 to water the refuse, because it is against the goal of the method of D2.

      Delete
    4. But beware that you then do not add "and/or by draining moisture from the refuse" to the claim!

      Delete
  38. I get that a "water container" per se is novel over a "tap" per se (i.e. in isolation). However, I disagree that "connected to a water container" is novel over "connected to a water tap". A water container encompasses anything that contains water. A "water" tap is implicitly connected to something that contains water. Otherwise how does it work? This really threw me in the exam and I spent a lot of time trying to decide what to do. I ended up rushing through the response as a result.

    ReplyDelete
    Replies
    1. A "water" tap is in my opinion not implicitly (i.e. inevitably) connected to a container. The water tap could also be connected to a lake or dwell etc. But I also had to struggle during the exam whether this feature would indeed be novel over D2.

      Delete
    2. But then you could argue the lake is a water container...

      Delete
    3. seems far-stretched that a lake is a water container.

      Delete
    4. I would hope that the word 'tap' was used instead of 'container' on purpose to differentiate them (at least because that's what I was relying on). If this wasn't supposed to be difference then I think the EPO would have simply called it a container instead of a tap

      Delete
    5. I guess there is a question mark over how to construe a water container. To me there is no indication that it should be construed narrowly.

      Delete
    6. I think that the problem is basically, where the line between applying general knowledge and semantics and "disallowed" technical reasoning is drawn.

      In real life I would totally agree that a water-spraying device connected to a tap would also be connected to a water container (at least some form of container).

      In the exam, though, you may and must apply general knowledge and semantics, but you must not apply any technical reasoning. There was no disclosure or defintion in the paper that a tap and a container are the same. Technical reasoning is not allowed.

      In any case, I think that there is a serious question of whether a water container is inventive over a water tap. There is simply no explanation of any technical effect of the container. This paper was simply badly drafted. Usually you would expect either some effect of the container making it inventive or some sentence such as "instead of a container, any other source of water may be used", which would make it clear that the container is not inventive over the tap.

      Delete
  39. Dear DP,
    I would be interested to hear your opinion about purpose-limiting claim 1 in the preamble to container for composting organic refuse by earthworms, in order to overcome D2 as being not a suitable container for the composting with the light sensitive earthworms. After all, the container in D2 has a light source and a transparent lid. I was really tempted to use this amendment, but then decided it is not safe, because the light source does not have to be switched on all the time. If the light source is switched off and the container of D2 is placed in the dark, it would still be suitable for composting the refuse with the earthworms. I really regret that the description of D2 did not give any clarifying details on this account. The whole purpose of the light source in D2 seems to be very unclear.

    ReplyDelete
    Replies
    1. I also tried to limit claim 1 based on such a "suitable for" to be novel against D2. Basically, I used the formulation from the very first paragraph: "compost container for producing organic fertilizer from organic refuse using earthworms" (so also changing "container" to "compost container").

      I interpreted the wish of the client to delete the original "for organic refuse" as hint that one should do something with this part.

      Furthermore, there were quite some hints that D2 is not suitable for this purpose, which I then cited under the novelty section, which really made me feel confident that this should be the right way of moving forward during the exam:
      * D2 does not disclose a compost container, but a container "to produce a quality nutrient from housefly pupae", as in D2, title
      * D2 has the transparent lid and the external lightsource (D2, [0001] and [0003]), but earthworms do not like it (D1, [0001]).
      * Only earthworms can be used to produce compost, but not other insects like the ones in D2 (somewhere in D3)
      * From the table in D3 you can derive that the reducing of humidity in the D2 container is quite deadly for earthworms.

      Would also be interested what others think about this to gain novelty over D2.

      Delete
  40. Dear DeltaPatents-Team,

    Have you considered the amendment "non-transparent lid"?
    This amendment
    -immediately eliminates D1+D2
    -provides in [013] a technical effect vis-a-vis D1 as CPA,
    -allows to formulate the problem based on the the effect in [13] and further on [7] as "provision of a container, which allows the control of moisture in organic refuse in the container"

    Furthermore, there seems to be no drawback for "non-transparent" in the description/client's letter.

    ReplyDelete
    Replies
    1. Yes, for me that also is still the most elegant solution. The earthworms anyways like it dark and since you anyways could not add the flies to claim 1 there is no drawback.

      Delete
    2. However, if the container is located in a dark area, the covering lid may be transparent [13]

      Limitation to a non-transparent lid will eliminate a commercially relevant embodiment from the claims.

      Delete
    3. Not true Anonymous March 07, 2021 7:20 pm. What's the use (commercial or otherwise) of having a transparent lid? No such use AT ALL is shown. In a dark area even with a transparent lid you won't be able to see anything. So no use. Eli+Roman is correct

      Delete
    4. (I am anonymous of March 7 - 7:20pm)

      There is no use of having a transparent lid.
      But that is not the point.

      The point is that a container having a transparent lid can still be used in a dark area. Therefore, there is a market for a container with a transparent lid, although it is limited.

      Imagine a costumer, who wants to grow worms and wants to buy a container and intends to set it in a dark area. This costumer will not care about whether the lid is transparent or not. Our client may have a patent on non-transparent lids, yet he cannot stop his competitor from selling a transparent lid to the costumer for lets say a very low price.

      That was my point, you are loosing relevant scope of protection when limiting to a non-transparent lid. That is the drawback.

      D1 teaches in [1] that the container must be kept in a dark area. D3 teaches a non-transparent container, but no technical effect/advantage. In real life, I would argue that is obvious to provide a non-transparent lid to the container of D1, since D1 give inducement for modidying the material and D3 provides a suitable material in the framework of growing earthworms. In paper B life, however, I would not be so sure about inventive step, since D3 does not provide any teaching for the technical effect of a non-transparent lid.

      Delete
    5. According to my reasoning, the non-transparent lid is only used to achieve novelty over D2. For the inventive step it is not really relevant anymore. I even put it in the preamble since D1 is CPA and D2 teaches away from the other "moisturization" features.

      Delete
    6. Imho the non transparent lid may be used to argue in the PSA that the skilled person starting from D1 would not consider a combination with D2 as D2 explicitly teaches to use a transparent lid and even a light bulb. For me, this was an indicator to limit claim 1 to a non-transparent lid.

      Delete
    7. (anonymous from above)

      I do agree that the non-transparent lid may be inventive.

      My only point was that it is rather narrowing and there may be a better solution for overcoming D2 that is less detrimental for the scope of protection.

      Delete
    8. @AnonymousMarch 07, 2021 9:50 pm It doesn't matter what the scope of invention is if the claim is not novel and inventive
      Eli, see my long point below (towards the end or search for my name) according to which non-transparent lid may also be used for inventive step: If the technical problem is defined something like: producing an optimum environment for earthworms to thrive.

      Best,
      Rameez

      Delete
    9. Unknown from above.

      In my opinion the limitation to a non-transparent lid was more acceptable than the limitation to a water container.

      Moreover, the replacement of a water tap by a water container is notorious for a skilled person and, thus, in my opinion may never constitute an inventive step!

      Therefore, I believe the above solution of DP may not get the job done.

      Delete
  41. Am I the only one considering the feature of "earthworms" as optional? The description in par 1 clearly states "worms, in particular earthworms". Based on such statemement, I replaced earthworms with worms even in claim 1. The expression "in particular" clearly points to an optional feature (see also Guidelines F VI 4.9

    ReplyDelete
    Replies
    1. Did you catch this passage of D3:
      [001] Earthworms belong to the class of worms, which comprises many different
      distantly related animals which develop in a plethora of substrates.

      Delete
  42. I think that for claim 1, it is most relevant to add some feature that distinguishes it from D2.

    Original claim 1 was not novel over either D1, D2 or D3.
    The client added a water-spraying device. This makes the claim novel over both D1 and D3, which do not disclose such a device.
    Usually, the claims amended by the client show the direction for inventive step. Adding the water-spraying device makes claim 1 inventive over both D1 and D3. D1 teaches the removal of moisture, D3 teaches keeping moisture constant, but the water-spraying device allows addition of moisture. Spraying water on the refuse will add moisture, where the temperature is high or humidity is low (par. [5] of the description). There is no suggestion in the prior art of adding water to refuse. The claims as amended by the client are inventive over D1 and D3.

    Problem, however, is that they are still not novel over D2. The added feature of a water-spraying device was also disclosed in D2, so the claim is still not novel over D2. I think the water container needs to be added to comply with Art. 123(2). The water container will make the claim novel over D2. However, it will not confer inventive step. The only difference over D2 is water container instead of water tap, it is merely an alternative way of providing water, no technical advantage.

    Problem here is that the container is still not really limited to organic refuse/growing earthworms. I think it is key to add another feature that clearly distinguishes the container from D2, that makes D2 totally unsuitable as CPA and shifts the technical field completely away from D2.

    I assume that the best feature to add is that the drain holes prevent earthworms from falling from the upper compartment into the lower compartment. It will solve all problems with D2. In D2 the larvae need to fall into the lower compartment. The feature wil therefore make the container completely incompatible with D2. Furthermore, it is not really a relevant limitation, since for growing of earthworms and producing of fertilizer, it is essential that the earthworms do not fall through the holes.

    D1 is then the only suitable CPA, it will not be combined with D2, since D2 has larger holes and also has the light source and the transparent lid, and the water-spraying device in D2 has a different purpose and D2 teaches the removal of moisture, not the addition.

    The holes need to be kept in the claims. With respect to the lower compartment, I would go with removing it altogether, based on [18], which implies a container, where the lower compartment has been removed and is placed on soil. Such a container has no lower compartment, therefore it should be clear that all the other features of the container are disclosed without the lower compartment.

    Otherwise, I would simply keep the lower compartment as in the original claim. Adding that the lower compartment is removable adds an unnecessary limitation. In the original claim, the lower compartment could be removable or not removable, it was not specified. By adding that is is removable, a competitor can add a compartment that is not removable and circumvent the claim. Defining the removable as optional, does not limit the claim and is rather the disguised addition of a sub-claim.

    The housefly eggs need to be removed for lack of disclosure.

    ReplyDelete
    Replies
    1. This is the solution I went for.

      Mich

      Delete
    2. Bingo I like it. I did the same but had the moisture detector in claim 1 to confer novelty over D2 instead of the holes not letting earthworms through. I think yours is better.

      Delete
    3. I did not find this solution in the exam, either. I arrived at the point that I need a feature to delimit from D2, but did not manage to find the feature with the holes and the worms.

      Instead I defined that the container "comprises" organic refuse with earthworms, which I now think is less than ideal, since it is somewhat blurry, whether chicken manure may also be considered organic refuse and additionally it is a significant limitation, whereas the holes not letting worms through seems to be more or less irrelevant for the scope of protection, because otherwise, the container will not work for the purposes of the client

      Delete
    4. Hi TT, I had EXACTLY the same logic and ended up with the same claim 1 as you did.

      Delete
    5. Also I found the remark from the examiner that the holes in D2 were suitable to drain water was a hint to specify that the holes in claim 1 could also prevent the worms from falling

      Delete
    6. Here is what I proposed in my paper:
      A container (1) for organic refuse (7), comprising
      a. an upper compartment (3) having a support (3a) for holding the refuse (7) populated with earthworms (8);
      [b. a lower compartment (4) for collecting excess moisture (6);]
      b. [c.] drain holes (5) permitting moisture to drain from the upper [to the lower] compartment and preventing earthworms (8) from falling from
      the upper compartment (3); and
      c. [d.] an upwardly opening covering lid (2) which fits over the container (1),and
      characterised in that the container comprises a water spraying device (13) connected to a water container (14) for adjusting the moisture.

      Delete
    7. The water container is clearly enough to be new over D1-D3. And in real life you would go for an inventive-step-argumentation based on D1 as CPA. No way that D2 ist CPA. Therefore I consider the water container to be enough for i.s. either.

      For the functional feature of the holes, I would not expect this to be the model solution since it is not clear under Art. 84.

      The DP solition can't be correct with respect to the optional removabel lower compartment - so I fully agree with your above analysis with regard to this point.

      What do you think?

      Delete
    8. @Thomas, I would not be surprised, if your solution comes close to the model solution.

      instead of the drain holes preventing earthworms from falling, I replaced "for holding" in feature a. with "comprising"...assuming that organic refuse and chicken manure are different things (it was not really defined in the paper, although the table of D3 seems to suggest so), I think this will also distinguish over D2 and make D1 clearly CPA, however, it is certainly more limited than your version and thus far less ideal (also sort of goes against the wishes of the client)

      Delete
    9. @TT: I do not think adding the limitation that earthworms should be prevented from falling through the drain holes is what you should have been looking for.
      First: this is already implicitly disclosed by the feature of the support (3a) which holds the refuse with earthworms, otherwise it would be only holding the refuse without holding the earthworms. The support as a whole is holding, regardless of drain holes being present or not. This of course implies that <hen you add drain holes, these should be sufficiently small in size.
      Secondly: document D2 describes an apparatus which separates larvae from manure (first sent.) and further that manure also encompasses mixtures with worms (par. 2). Hence, D2 teaches also separating larvae from worms in general, and earthworms in particular.

      Delete
    10. @BS: I do not fully agree with your first point. I think the supports "holds | the refuse with earthworms", I would not derive from this that it holds/retains the earthworms as such and implies that the earthworm do not fall through the holes.

      I agree that you second point is noteworthy. I understand you argument that D2 would implicitly disclose that the mesh would not allow for migration of worms? I think there is some point there that it would fit a purpose of separating larvae from manure/worms.

      However, I think there is no real technical information in the entire paper about the relationship of the worms and larvae, which makes it very hard to assess, whether D2 has this disclosure or not. From a tactical point, however, and when thinking in a simplistic way, the mesh of D2 is certainly configured to let other things than water pass, whereas the "drain" holes of the application only have the purpose of draining water.

      Which feature would you use for delimiting claim 1 from D2. I do not think that features from the sub-claims should be used, as these are denoted as non-inventive in the Office Action and I think the Office Action has to be taken for granted in paper B.

      Also, the wording of the Office Action should be noted. It does not merely raise an objection under Art 56 to the sub-claims, it says "Dependent claims 2, 3 and 5 do not appear to contain any features that could
      contribute to an inventive concept"

      I understand this as a hint that the features of claims 2, 3 and 5 do not contribute and should not be used for inventive step.

      However, then spraying water (claim 5) would also not contribute....hm...this paper is just totally confusing. I am really curious about seeing the model solution ;-)

      Delete
    11. @ Anonymous March 07, 2021 8:51 pm
      I don't think the functional feature "preventing from falling" is unclear. It's the same as "permitting moisture to drain" which was already present in the claims and not objected in the Art 94(3) communication. Those two features are found in the same sentence in A1.

      Delete
    12. I followed the same approach and my claim looks almost identical to the one posted by Thomas above. It's comforting to see that I am not the only one took this approach.

      Delete
  43. Imho, for the container with holes, it must be specified in claim 1 that they prevent worms from falling lower, since this is an essential feature. It does not matter that the Examiner has not made an Art. 84 objection yet: claim 1 must be amended to satisfy Art 84 in any case.

    ReplyDelete
    Replies
    1. I agree that addition of this feature solves most problems and I currently think it might be the best way to go, maybe even the intended solution (see my post above)

      However, I do not think that you need to adress issues that were not raised in the Office Action in paper B. In paper B, the Office Action has to be taken for granted, it is always correct.

      Additionally, from a real-life perspective, the feature "preventing worms from falling into the lower compartment" presumably lacks clarity itself and I would expect a real examiner to raise an Art. 84 objection to such a claim. What size need the holes to have? At what size do worms fall through?

      That would be one of the problems that speaks against making this limitation, even in the context of paper B, because additions from the description need to be checked for clarity.

      Delete
    2. The water container is clearly enough to be new over D1-D3. And in real life you would go for an inventive-step-argumentation based on D1 as CPA. No way that D2 ist CPA. Therefore I consider the water container to be enough for i.s. either.

      For the functional feature of the holes, I would not expect this to be the model solution since it is not clear under Art. 84.

      The DP solition can't be correct with respect to the optional removabel lower compartment - so I fully agree with your above analysis with regard to this point.

      What do you think?

      Delete
    3. Why would D2 not be considered as CPA?

      For the claims of the client, it is novelty-prejudicial. By reincorporating "for organic refuse", D2 would still be novelty-prejudicial, since this feature is interpreted as being suitable for organic refuse and D2 is suitable.

      By adding the water container, it becomes novel. But why shouldnt D2 still be CPA. The only difference would be the water container. The feature "for organic refuse" does not change the CPA. The claim is to a container suitable for holding organic refuse populated with earthworms. D2 also discloses a container suitable for holding organic refuse populated with earthworms. It is the same field, D2 has the most structural features in common. The difference is merely the water container.

      The feature "suitable for something" cannot shift CPA away from some document, which is suitable for the same purpose without mentioning it. Otherwise it would be very simple to obtain protection for simple variations by simply defining a suitability in the claims, which is not really limiting, but not mentioned in the prior art in question.

      I think you need to add a feature that disables D2 as CPA.

      Delete
    4. (anonymous of 9:05pm-March 7 was me, published as anon accidentally)

      Delete
    5. PS

      If subject-matter is obvious when starting from some document as CPA, there is strong indication that this document is indeed the "correct" CPA.
      If it not obvious when starting from a different document as CPA, there is even stronger indication that the first document is indeed CPA. After all, it clearly seems as the most promising starting point as you arrive at the claimed subject-matter without inventive step.

      Delete
    6. I think the choise of the CPA should be done based on the purpose, not based on "suitable for". "Suitable for" is the correct approach for novelty.

      Delete
    7. And paper B is designed to have only 1 CPA and this is clearly D1 even if you only have the water container as distinguishing feature over D2 in terms of novelty.

      Starting from D1, you would agree that the claim is inventive?

      Delete
    8. Yes, starting from D1 it would be inventive, I totally agree.
      Starting from D2 it is not inventive.
      Therefore, D1 cannot be the closest prior art, because D2 is apparently a more promising starting point. From D2 you can arrive at the subject-matter without inventive step. From D1 you need inventive effort. How can D1 be the CPA? It is apparently further removed from the subject-matter.

      If you start from the device disclosed in D2 and replace the tap by a container, you arrive at a device that falls within the definition of claim 1. You would gain protection for some trivial modification to a known device, merely by defining a different purpose of the same device. However, the purpose defined in the claim does not limit the claim (only insofar as suitability, but that would apply to D2 also). If the claim was a use claim, the situation would be totally different.

      The CPA is the most promising starting point for arriving at the claimed subject-matter. Having a similar or the same purpose is a strong argument. However, you have to define the purpose based on the structural and limiting features of the claims, not what is in the description or what the intention of the applicant/inventor is. If the claims is broad, then its purpose is broad.

      As pointed out before, Claim 1 pertains to a container for holding organic refuse with eartworms. D2 also pertains to such a container. It has the same purpose.

      Delete
    9. TT, Re "However, I do not think that you need to adress issues that were not raised in the Office Action in paper B. In paper B, the Office Action has to be taken for granted, it is always correct":

      I am sure that there is no rule that response should be only to issues raised in the Communication, from trainings and due to IPREE, Rule 24(3) stating that claims must be amended as appropriate to meet the requirements of the EPC.

      Also, while there is a presumption that the objections and comments in the Communication are correct, sometimes there may be mistakes, and a part of the job of the patent attorney is to detect them (you can find such a statement in the EPI Report for one of two last EQEs).

      With regard to "Additionally, from a real-life perspective, the feature "preventing worms from falling into the lower compartment" presumably lacks clarity itself": I am not sure, but I do not think that this is relevant.

      IMHO, the only question is: does the Examination Board consider this feature, for the subject matter with drain holes, to be essential, or not? If yes, it must be in claim 1.

      Delete
    10. You are right, if it would be essential it would have to be in the claim.

      Putting some limiting feature into a claim to overcome some potential clarity objection that was not raised by the examiner is something you would never do in practice.

      I did not do all of the old B exams, but I think if they expected us to add essential features to overcome clarity issues that were not raised in the Communication, this would be the first time. However, there were many things in this B paper that have never been there before, so who knows ;-)

      But nevertheless, as you point out the claims must meet the requirements of the EPC. When adding stuff from the descrition you have to check for potential clarity issues. I do not think that you can assume that everything that is written in the description would be clear, if added to the claim.

      Although I think that the feature is very good choice for overcoming the issues with respect to D2, I still have some reservations with respect to clarity of the feature itself.

      And with respect to Rule 24(3) IPREE, it fully states
      "(3) Candidates are expected to respond to all points raised in the official communication. The response shall be in the form of a letter to the EPO accompanied by the claims supplied by the client, amended as appropriate to meet the requirements of the EPC."

      I could be read as meaning that the amendments should relate to points, which have been raised in the Communication. Although, it could also interpreted differently, I grant you that.

      Delete
    11. The last "anonymous" was a different one, here is my reply:

      I still cannot agree:
      According to G VII, 5.1 and also according to what CEIPI/DP teach, you have to make a quite formalistic choise of the CPA, starting from the technical field which should not be defined too wide (because the skilled person would always put forward the specific technical field). The next criterion is the purpose and the last (and often even not necessary to check) criterion are the stuctural/functional features.

      Usually, in the exams, they want to test the candidates, if they get trapped by directly focussing on the features ignoring the technical field and the purpose when chosing the CPA.

      Here, in my opinion, this was tested very staightforward - why else did they chose D2 to have a tap and claim 1 to have a water container, while not mentioning any advantage/technical effect of these features? The candidates were tested if they see that D2 is not CPA, even though D2 has the most similar features.

      In reality, you may have a nice and lenghthy discussion with the examiner, if the PSA should be conducted twice: Starting from D1 as CPA and from D2 as CPA. But in Paper B, it there is "the one and only" CPA, which must be D1 because auf the first two criterion as mentioned above. This is kind of an "exclusion precedure" chosing the one and only CPA which stops at criterion 2 (purpose = composting based on organic wast and earthworms).

      Also, from the perspective of exam design, is does not fit into the Paper B approach of the exam commitee (at least until now) that you have to search for features which are absolutely not mentioned by your client. In particular, there was no hint at all directing to the feature of the size of the holes that the earchworms do not fall down. And apart from that, this feature was not defined as to comply with Art. 84.

      Delete
    12. Finally managed to reply with name instead of "anonymous". Sorry for the confusion, TT.
      I'm the "anonymous" who wrote the whole stuff apart from the one comment.

      Delete
    13. This is the "different one" :)

      RE: "In particular, there was no hint at all directing to the feature of the size of the holes that the earchworms do not fall down":

      For me, the hint was there, since you arrive at this feature while considering what to do with holes (remove or not to remove).

      Moreover, if the Examination Board keeps holes in claim 1 and (a) mentions this feature as inessential or (b) does not include it into claim 1, an appeal may be filed. It would not be reopening of the marking procedure of some candidate, but would be analyzing the Examiners' report v. the application.

      Delete
    14. The point is that the purpose of claim 1 as amended by the client and with the water container is not limited to organic refuse/earthworms.

      The original is not novel over D1 and D2. It has the same purpose as D1 (it is the same device). It also has the same purpose as D2 (it is the same device).
      If you add the water-spray, you are still not novel over D2. Thus, you have moved the subject-matter of the claim closer to D2 and away from D1. Adding the water container makes you novel over D2, but does not provide anything additional with respect to the purpose organic refuse/earthworms. It is a feature that is completely compatible with D2 and an adaption of D2 that could clearly be made to D2 without being contrary to the purpose of D2. The point is that the distinguishing feature of a water container does not change the purpose of your device towards D1. It does not go against the purpose of D2. The device may still be used with chicken manure for growing housefly eggs. It is still a device with the same purpose as D2.

      My point is that the feature providing novelty of D2 also clearly needs to shift the purpose of the device towards the purpose of D1 and away from the purpose of D2. It should go against the purpose of D2 and provide a modification of D2 that a skilled person would not make. Otherwise D2 may still be considered CPA as the claimed device is still suitable for growing your larvae in chicken manure. (it has both purpose or all ourposes for which it is suitable, just as the device of D2).

      (To make an anology with this years paper A. Adding the columnar microstructure provides inventive step over D1, but the claim is still not novel over D2, unless you specify the thickness of 25µm. Here, the distinguishing feature of the thickness does shift the purpose towards the subject-matter of D1, because the feature providing novelty over D2 is technically related to the different purpose of D1, providing thermal insulation, the 25µm is a modification the skilled person would not consider making to D2 and therefore this feature rules out D2 as CPA).

      The Guidelines litteraly state in G-VII-5.1
      "The closest prior art is that which in one single reference discloses the combination of features which constitutes the most promising starting point for a development leading to the invention. In selecting the closest prior art, the first consideration is that it must be directed to a similar purpose or effect as the invention or at least belong to the same or a closely related technical field as the claimed invention. In practice, the closest prior art is generally that which corresponds to a similar use and requires the minimum of structural and functional modifications to arrive at the claimed invention".

      It must be directed to the same purpose or effect or at least be in the same technical field. D2 fulfills this criteria. It is in the same field and the claimed container can be used in the same way as the container of D2 and therefore has the same purpose.

      The closest prior art is that which in one single reference discloses the combination of features which constitutes the most promising starting point for a development leading to the invention. That is the definition of the closest prior art. It is not the document with the most similar purpose. The combination of features in D2 would be the most promising starting point, since it requires a trivial adaption from a water tap to a water container. There is nothing that would prevent the skilled person from making that adaption. D1 is a less promising starting point, because it requires more adaptions, which are less apparent to the skilled person.

      How can a claim that is not inventive when starting from D2 meet the requirements of the EPC?

      (Have a good night though ;-))

      Delete
    15. It is often the case that a claim is only inventive from one COA, but not from another. This is why we have the step of choosing the CPA. Otherwise, we would not be in a PSA (this would rather be the approach which is condicted by the german patent office which do not choose a CPA).

      Under the PSA you have to make this choice and the purpose is absolutely not the same as "X is suitable for".

      To make an example:

      Claim 1 refers to a piece of cloth for use as a towel with a specific surface structure. The purpose is, according to the description or explicit wording in the claim, to dry oneself with it.

      D1 is a normal towel. D2 is a bed sheet with the same surface structure as claim 1.
      Now, claim 1 is not new compared to D2, because a bed sheet falls under the generic term piece of cloth and is also suitable for drying oneself.

      However, if one were to examine inventive step, one would have to start from D1. (technical field and purpose) The claim is therefore inventive, although it is not novel. So say it more clearly: For inventive step, it is not relevant, whether D2 is "suitable for".

      This is exactly what distinguishes the PSA applied by the EPO from the approach of the GPTO.

      Delete
    16. So if you add something to claim 1 which makes it new ober D2, younhav a novel and inventive claim.

      Thanks, good night to you :)

      Delete
    17. @Gereon
      The problem-solution approach, strictly speaking, does not require the step of choosing the CPA. You can apply it to any document at hand. The choice of the CPA is a useful administrative tool which is designed to avoid the overhead to have every document examined to the substance. In the case law book, you have decisions/explanations which state that it is the examining division which selects the CPA, not the skilled person, and that this choice is thus an administrative one. Furthermore, the case law (also described in case law book) repeatedly says that any document can serve as CPA provided it is a promising starting point for an IS attack. If your invention is inventive over the CPA, it should be inventive also over the other documents. If it lacks inventiveness over another document, then the other document should have been selected as CPA.
      This also holds for your towel example. The piece of cloth with the specific surface structure lacks novelty and for IS one could still use the bed sheet because it is suitable for drying oneself and it is cloth. In this case it would be better to just claim a towel, because being a towel is not the same as replacing a towel.

      Delete
  44. What basis under A123(2) EPC would you give for adding the backreference of independent method claims to the preceding container claims? In what way is "mental act" claim 6 unitary (A82 EPC) with the device and method of claims 1 and 4? Even though the examiner's report does not raise the objection, but shouldn't 2 independent claims in the same category (method) be justified why do they comply with R.43(2) EPC requirements? Furthermore, aren't there essential technical features missing in claim 6 for carrying out the method (see Guidelines section under clarity of CII)? Thank you.

    ReplyDelete
    Replies
    1. I did remove the backreference of the method claims, since I also found it questionable, whether there was disclosure and I also considered them unnecessary

      I did not provide any arguments wrt Art.82/R.43(2). However, I would argue that all claims have the common feature and common inventive concept of "adding" moisture to refuse, which confers unity of invention. Claims 4 and 6 are alternative solutions and thus allowable under R. 43(2).

      Delete
    2. Hi Anonymous, I like your thinking very much. I could not justify a backreference of the calculation method claim to the device claims. So I deleted the reference and introduced the infrared measurement to make it technical.
      That was the only solution I could find to justify IS but I was also thinking that my claim was lacking unity. Now your argumentation that unity is met based on the moisture adjustment makes sense :)

      Delete
    3. I also felt that by removing the backreference from claim 4/5, the client would get some protection for alternative ways of moisture removal, since claim 4/5 is not limited by the drain holes.

      Delete
    4. Although claims 1 and 4 adjust moisture by spraying water while claim 6 merely adjust moisture...

      Delete
    5. "recommending adjusting the amount of moisture added to the refuse"

      It adds moisture. Spraying water adds moisture as well. D3 keeps moisture constant, D1 and D2 lower moisture.

      Delete
  45. I would also like to express my regret that this new online browser configuration was not taken into account for paper B. On-screen reading of important parts of the assignment, the need to jump back and forth separate documents within one big single PDF file, associated with very limited editing options was quite cumbersome and took a lot of extra time. The 3.5 hours given were too short considering the extraordinary difficulties. It should have been realized that 7 pages of application docs including 3 independent claims with numerous amendments by the client would require an extensive added subject-matter assessment. The fact that you could print the 4 prior art documents (3 + TPO) was welcome but couldn't compensate for the longer time needed for checking the application for amendments by analysing the client's letter in combination with the client's amendments, one at a time on a single screen. The time lost could not be used for the more important inventive step reasoning, which is considered the core of the exam. It was probably not the best choice to increase the difficulty of the added subject-matter part this year, when already the task of navigating through different parts of a document on a screen is an obstacle in itself.

    ReplyDelete
  46. I read paragraph [005] of the spec as background disclosing the use of a sprayer to adjust the moisture was known in the art. The spec reads like background until [007].

    ReplyDelete
  47. Any comments on the CPA and other issues? It seems the committee this year will need to accept multiple solutions if the solution is well argued under Art. 123(2), novelty, and inventive step. I think the compensable fail rate will be higher this year.

    ReplyDelete
  48. This amendment to claim 1 "b. a lower compartment (4) for collecting excess moisture (6), wherein optionally the lower compartment (4) is removable," is strange because you claim the lower compartment and then you say it is removable. That automatically narrows the claim and goes against the client's wish, but I understand your point about Art. 123(2). I have never seen a claim in real life or in the EQE were you claim a component and then say the component is optionally removable. It seems if the client wanted the broadest coverage, he/she would not claim the lower compartment? The lower compartment is not related to novelty, but more related to Art. 123(2). If there is direct basis for removing it (there was in application), it is unreasonable to try removing it. Moisture can leave in many ways (other than drain holes), e.g., evaporation, chemical reaction with soil, temperate can change, the lid can be opened, etc. It is not necessary the claim cover all of those, only the essential elements are necessary. I don't have the feeling the lower compartment is essential.

    ReplyDelete
  49. Dear DP,
    All other things being equal/also-added, I think you may have missed a most elegant solution which is the provision of a non-transparent lid. No advantages are shown for a transparent lid in the application. And no drawbacks for non-transparent lid.

    On the contrary, only advantages have been disclosed for a non-transparent lid. All through the application we keep finding hints as to why a "dark" environment is so great for earthworms. Para 013: "The covering lid (2) is preferably non-transparent to achieve the dark environment the earthworms need for optimal development." And Para 015 reiterates why dark conditions are important: "This helps to produce the moist, dark conditions preferred by the worms."

    D1 itself discloses: "Earthworms are light sensitive and so the container needs to be kept in a dark area." So the container of D1 ALWAYS needs to be kept in a dark area. It cannot be kept in an area with light. This is solved by the container of amended claim 1 which has a non transparent lid and this container on its own, without needing to be placed in a dark area, provides the moist, dark conditions that are optimum for earthworms.

    Therefore, I believe having a non-transparent lid, all other things equal/added, is a straightforward and elegant solution that is begging to be implemented.

    ReplyDelete
    Replies
    1. Dear Johan,
      I appreciate the non-transparent lid solution, it has definitely some merits.
      The question is whether you can defend it against an IS attack based on D2 as CPA and common general knowledge. D2 has a transparent lid, and earthworms are light-sensitive. A non-transparent lid has the effect that the container can be used outdoors too. Would the skilled person consider making the lid opaque when the goal is to use the container outdoors for the purpose of composting organic refuse in the container, using the light sensitive earthworms? In my opinion, he would consider this option and that's why I did not go for it.

      Delete
    2. Dear BS,
      I appreciate your answer. But disagree strongly. Let me explain. Granted, it is common general knowledge that earthworms are sensitive to light. But that doesn't matter. It would only matter/be-problematic if the common general knowledge was disclosed as follows: "It is generally known to provide a non-transparent lid for achieving dark conditions in such containers". Or something of that sort.

      Thus, without the above (or some variant of it) being disclosed as common general knowledge, what you are suggesting is sheer hindsight.

      Best,
      Johan

      Delete
    3. I do not see how D2 could be CPA for his/her claim 1 at all. D2 has no earthworms and the goal is to dry the refuse/feces.

      Delete
    4. Dear AnonymousMarch 07, 2021 10:05 pm
      I fully agree with you. I did not bring this point up in my response to BS as I thought it was more important to answer his point about about common general knowledge.

      Delete
    5. @ Johan
      I agree that D1 is the more logical choice of the CPA, but in real life your invention can be tried for IS against any document. Also in past B papers, IS is often discussed with respect to more than just a single document of the given prior art. So even if it is not CPA, let's just assume it is, only for the purpose of analysing IS with respect to D2.
      D2 does not mention earthworms specifically, that is true. But it is also true that it mentions a support that retains worms in general (thus including earthworms) and lets larvae pass. The container of D2 is thus also has a compartment that is fit to hold organic refuse with earthworms.
      What is then the feature of the invention that distinguishes it from the container of D2: the non-transparetn lid in your case. The effect thereof is that that the refuse with worms is kept in the dark.
      The skilled person asked to modify the container of D2 such that the worms hold by the compartment remain in the dark, would he then not consider changing the tranparetn lid into a non-transparent lid, especially if D2 mentions that the tranparent lid lets light shine into the compartment?
      I guess the way the problem is stated now does not involve hindsight, but I would like to hear your opinion. How would you state the technical problem to be solved when you rely on the non-transparent lid as distinguishing feature?
      This is only me challenging you. Personally, I am on your side and also of the opinion that the lid of D2 should remain transparent because it is disclosed together with the external light source. I really really regret it that the paper was not drafted in a way to include more indices or explanations. In the case of the light source for example, one could have included that it is absolutely necessary to also have light shining into the container of D2 also at night, to make sure that the eggs hatch (I am not sure whether this is true, but something of this kind would have helped a lot).

      Delete
  50. So the correct argument is that a tap which contains water is not a container of water?

    I would consider that borderline at best (and I would like to see the reply from an Examiner if I tried that in real life!) If I turn a tap off there is water contained in it. Did I miss some definition of a container that would exclude a tap?

    ReplyDelete
  51. "Even among DP's experienced attorneys and tutors, this year’s paper led to dramatically different solutions."

    What a damning verdict on this so called examination.

    We could spend the next three months arguing what the "correct solution" should be, but I think it's time we acknowledge what an inadequate test this was. The paper should be an examination of a candidate's fitness to practice. If DP's attorneys can't agree and then post a rather unconvincing consensus claim with optional features, it's little more than a lottery. I recommend everyone take another look at the 2019 paper and remind themselves what a typical hard paper B looks like.

    This year's paper wasn't hard, it was just illogical. It looks like an exam that tested poorly and then changes were made (which made things worse) and then it was not retested. When I compare it to the intricacies in this years paper C, it's incredible. Hardly any feature was disclosed with more than a single sentence. The spraying device was half heartedly disclosed in two sentences with what appears to be background knowledge indicating spraying was known.

    The invention is supposedly: "[007] The purpose of the present invention is to provide a container, which allows the control of moisture in organic refuse in the container." Lets face it, the sprayer and holes on their own don't provide this. There should have been at least one paragraph describing a spraying device responsive to a moisture sensor, an associated technical effect of moisture control and a clearer steer from the client or an added matter/A84 necessity. We could all have unambiguously hung are hat on such a feature. But alas no, I've read comments covering every solution from non-transparent lids to floating lids to moisture sensors to fly larvae to optionally removable lower compartments to the features of claim 6. Whatever the EPO's model solution, most of us will still feel justifiably conned.

    ReplyDelete
  52. This was a highly ‘interesting’ paper B indeed… I’m pretty sure I was very well prepared, but this paper made me very disoriented. I practiced many training exams, I even had to train myself not to be too creative in making too many amendments in paper B, so I pushed myself to keep it simple. I now got the feeling that that was not the best choice for this year.

    I wanted to go with the client as good as possible and thought arguments could be found indeed to remove the features b (lower compartment) and c (drain holes). In my opinion par. [018] showed that the lower compartment was optional where it is said that "the lower compartment may be removable, allowing.." etc., still perfectly working invention without lower compartment. The drain holes were in my opinion also described as being not essential in par. [004] wherein it is said that “Excess moisture can be removed by, for instance, drain holes, water-absorbing material or by evaporation”. I read this as drain holes being one of the options for removing excess moisture; and hinted by the client in his letter by: “The skilled person knows that excess moisture can alternatively be removed by other means such as a water absorbent material.”.

    At certain moment, not yet comfortable about the claims, I felt –and I have the impression many candidates had that feeling– it was either going further down the rabbit hole and ending up with better claims but without any argumentation, or maybe the wrong claims but at least with a completed Art.123-84-54-56 argumentation.

    Best regards, KA

    ReplyDelete
  53. This comment has been removed by the author.

    ReplyDelete
  54. Honestly, i don't think this "removable" is a good solution. I simply deleted the element b and mantained the drain holes in element c and argued for the allowability. Introducing removable actually unduly limit the scope of claim 1, and it is definitely not the intention of the client....

    In this case, the client would like to have something with a larger scope, and if we introduce "removable", the scope would be even smaller than the original claim 1. I think in the past B papers, there is no such situation wherein we not only obeyed the instruction of the client, but even make the claim 1 worse than before....provided that there is no objection raised in this regard.

    Just my personal opinion.

    ReplyDelete
  55. It appears that during this discussion here, hardly anybody has raised the question concerning the alleged computer implemented claim would be novel and inventive over D3. I think it only is with the infrared lamp, there is also a hint in the application saying further means for determining the worm amounts are known meaning Infrared is not known.

    ReplyDelete
    Replies
    1. I think the way is to include the back reference to claim 1. This immediately solves all A.52(2), n+is, and unity issues. This was also hinted at by the client who said that the scope of claims as provided were sufficient for their business.

      Delete
    2. I think we all gave up caring by that claim and hoped it isn't worth many marks. In the last 10 minutes of the exam I just shoved some technical means related to water spraying in and said it was novel, unified and inventive for the same reasons as above. Probably less discussion because the answers for claim 1 are so varied and not obvious, and a lot of us just don't care/lost the will to live by that claim!

      I guess we will all live and die by claim 1.

      Delete
    3. I started with answering Claim 6 because I thought... "Oh, that's easy, just add a few technical features from paragraph 24 in order to make it more or less a regulating method." ;-)

      Delete
  56. Having read most of the comments and after having given it a final thought:

    The clients business needs relate to adding water to the refuse, by spraying. In claim 1, the spraying device is added, in claim 4 the method step of spraying is present, and claim 6 relates to calculating an amount of moisture to be added. Any of the technical steps of claim 6 (infrared etc.) would just be arbitrary without any instruction of the client to do so.

    Claim 1 as suggested lacked novelty over D2. The amendment suggested for claim 4 in my view made the claim inventive, because the only other way the skilled person knows how to add moisture is taught in D3: the lower compartment filled with water. The client however introduced the housefly eggs, which on their own in my view violate A123(2). In claim 6, we maybe should have added a step of adding moisture by spraying, which step has a technical character.

    But the real crux is in claim 1, and the lack of a clear and unambiguous hint in the letter of the client or in the description as filed, as to which feature should be added in order to make D1 the CPA, and which feature does not conflict with the business interests of the client.

    I will take solace in the fact that points are gained not only for the amended claims, but also for the arguments with regards to amendments, clarity, the TPO, and hopefully novelty and inventive step of a claim 1 which was not the intended claim 1 by the examination committee.


    ReplyDelete
    Replies
    1. I fully agree, what would you expect to be the intended claim 1 (so: is the water container enough?, See above discussion of TT/Gereon)

      I think TTs arguments are good and probably this would be the case in real life - depending on the examiner. But I think Gereon is right.

      Delete
    2. Honestly, I have no idea. One may (I think) validly argue that D2 also relates to a container for refuse and earthworms, and should not easily be disregarded as a CPA:

      D2 [002] The term “chicken manure” also encompasses mixtures of chicken manure with other biological material such as plants, insects, worms and/or soil.

      Plants are in my view comprised by the organic refuse from households of the application.

      If the water container was really the solution, and if I do not get 45 points or more, I will file an appeal to have the examination board explain to me how D2 does not disclose a water container. There has to be some source of water connected to the tap, otherwise you would not have a spray cleaning device. This source of water will always be some water container.

      I agree that we had to do something to make D2 not the CPA, since D2 is the only disclosure of the spraying device which is clearly suitable for the same purpose as in claim 1.

      Delete
  57. What a mess... They have to make at least 3 model solutions each giving full marks!

    My feeting tends to the argumentation of Gereon - the CPA is D1, so inventiveness (+). And the water basket makes it novel over D2, so novelts (+).

    In mechanics I had indeed often cases where you made a claim novel and then it was directly inventive because the former novelte destroying document had a different purpose.

    The purpose is the key of the psa and I can't agree with TT that beeinh suitable is the same as having the purpose.

    ReplyDelete
    Replies
    1. Thanks to the above discussion I came to the same conclusion. This will be the one model solution of epo but there will be awarded a lot marks for other solutions.

      Delete
  58. I think the exam comitee did not spot all the problems at all. It cannot be meant to have 10 different well arguable solutions. They have to do something about it..

    ReplyDelete
  59. What about changing "a water spraying device for adjusting the moisture" into "a water spray device which is configured to spray water onto the fuse, thereby adjusting the moisture of the fuse". As the cleaning device in D2 is definitely not used to clean the fuse (which will add moisture to the fuse which is not wanted by D2), such a claim will be novel over D2 and also D1.

    ReplyDelete
  60. By the way, I think using "removable lower container" in combination with "drain holes permitting water to drain from the upper compartment directly into the soil when the lower container is removed" makes the claim novel over D2, because making flying eggs dropping on the soil is definitely not desired in D2.

    ReplyDelete
  61. Hey Nico, Roel, Sander, and Jessica, thank you as always for the solution(s) and for trying to solve this year's Paper B mess!

    A question: What about a method for producing a fertilizer as proposed by you, but:
    (i) without referring to the container of the invention (where did you find suitable basis to refer the method to the container of the invention?...) and (ii) without adding the features "by spraying water on the refuse and/or by draining moisture from the refuse"? Would it be novel over D2 in your opinion?

    ReplyDelete
    Replies
    1. I forgot to mention also that: (iii) optionally from step b) was removed, whereby the step of adjusting moisture would be mandatory

      Delete
  62. Thanks all for your solution, it certainly is a really difficult paper this year. I have a few comments on your solution and what I ended up doing...

    I don't agree with the amendment to the lower container using the word "removeable". This doesn't seem to actually provide any useful change in scope to me. With this form of the claim, it seems that as soon as a competitor removes the lower container they would no longer be infringing... I felt when stepping through the Houdaille test that the description made it clear that the lower container is not necessary and can be deleted from claim 1.

    Also, I made the functional "means for removing excess moisture" amendment that you mention but dismissed as lacking basis. I felt there should be good support at least given EP GL F-VI, 6.5, since there is clearly a link between the holes and their function of removing excess moisture, and clear indication that there are other means for removing excess moisture. I was certain during the exam that this must be what the Examination setters must have intended given the hint in the client's letter saying that they felt holes were not necessary because other means could do the same function but they did not want to limit to an alternative means (i.e. the water absorbing material). As you say, this hasn't really been the type of amendment in recent B papers, but I have definitely seen an expectation for this sort of broad functional amendment in older EM paper Bs that I have tried.

    I also added the spray + container for A123 purposes, but I felt the stretch to say that the tap of D2 is not a container was just too much, so I also added the moisture detector. I found it interesting that the client deleted "display" from the dependent claim with no explanation in the client letter. I added this back in, but also added the alternative option of an alarm which, admittedly, was only disclosed in context of the method but I felt it showed that it was clear to the skilled person that an alarm was a clear alternative to the display when coupled with the monitor.

    I also amended both method claims to refer to the container of claims 1-3. I think the basis was a little unclear on this, but I felt that it would be clear and unambiguous to the skilled person that the inventive container could be used in both methods from the context of the application as a whole (e.g. because it has the particular means required for the steps of the method). In addition, I rea;;y don't think it would have been possible to deal with all the other issues in the paper properly and do more than one inventive step argumentation.

    ReplyDelete
    Replies
    1. i put the display also into the claim 1. However, now when I thnik about it, the deletion of display in the intstruction should be exactly a hint that we should leave it out when introducing the water spray into claim 1. It is disclosed in the application that such display is only optional.

      Delete
  63. I was puzzled by the examiner’s opening remark that the subject matter of the claims has been searched in its entirety. The examiner didn’t object lack of unity, however, the original claims are not unitary. So I thought that this remark, combined with the examiner’s statement that claim 3 and claim 5 (adjusting moisture by spraying water on the refuse) lack an inventive step and with the statement of the technical problem in paragraph [007], could be a hint that the new claims should all reflect the general inventive concept of moisture control, meaning both monitoring and active adaptation of moisture. This concept is already in original claim 6, so searched, only not yet technical. I therefore added both the moisture detector with a display and the water spraying device in claim 1. Claim 4 is amended by referring to claims 1-3 and claim 6 is made technical by adding the CII limitation. All independent claims then comprise same or corresponding special technical features. I furthermore left out the lower compartment in claim 1, and instead added that the drain holes are in the support (3a). In the meantime I regret this amendment. The suggestion in the original post is probably the much safer option. What a difficult paper B.

    ReplyDelete
  64. Is the feature "separating the earthworms from the fertilizer" completely useless? From the application this seems to be the purpose behind the method of claim 4 and the idea at its basis (simultaneously producing fertilizer and earthworms). Isn't it weird that the feature is nowhere considered?

    ReplyDelete
  65. I am a biologist and I would like to comment a bit based on my training experience.
    A lot of people are struggling wether D2 can be CPA. However, from my point of view, D2 can never be a CPA because it relates to something completely different from earthworm. Althoug D2 mentions something like worm (p.s. earthworm in this regard is novel over worm), a skilled person would never derive from D2 that the invention described therein can be credibly apply to worm.

    Thus, D1 -->earthworm, D3-->earthworm, and only one of them can be CPA. After introducing the water spray into claim 1, D1 should be closer to claim 1, because it describes a moisture detector, which is not described in D3. Hence, D1 is the CPA, and the distiinguishing feature is the water spray, and the effect of claim 1 is increasing moisture (or reducing the variation of huminity to the survival of earthworms). In this regard, D1 alone is silent in water spray, and only discloses drain holes to regulate the moisture. Thus, with the teaching of D1, a skilled person would most probably ajust the drain holes to regulate the moisture, but not introducing water spray. D2 leads away from the invention, because firstly it relates to something different and a skilled person would not sure whether it can apply to earthworm, and secondly a dry condition is intended, which is contrary to the present invention and would lead away from the claim 1. D3 discloses the effect of the moisture of the substrate on the earthworm. Thus, when combining D1 and D3, a skilled person would modify the substrate to regulte the mositure, but not introduce a water spray. Hence, claim 1 with water spray in this regard is inventive.

    ReplyDelete
    Replies
    1. Your explanation is very convincing. I believe that was the intended solution.

      Delete
    2. I agree 100%, see discussion above (TT vs. Gereon)

      Delete
  66. So going through all the comments again, and based on Paper B's logic and structure, here is my conclusion about what the expected model solution should have looked like more or less:
    1. A container (1) for composting organic refuse by earthworms, comprising

    a. a compartment (3) having a support (3a) for holding the refuse (7) populated with earthworms (8);
    b. a water-absorbent element or drain holes (5) provided in the support for removing excess moisture from the compartment,
    c. an upwardly opening covering lid (2) which fits over the container (1),
    wherein the container comprises a water spraying device (13).
    A dependent claim could have specified that the moisture removal is by draining it through the holes into a removable further compartment or directly into the soil.
    The generalization to means for removing excess moisture is tempting, but can be objected under Art. 123(2) since it would claim alternative not originally disclosed in the two figures (e.g. venting holes as in D1). I am not sure what case law says under which circumstances such a generalization would be allowed if more than one specific struncture has been indicated in the description...
    I am also not decided yet whether you could have retained the combination earthworm or earthworm and fly eggs in the claims. Personally I think there is enough support to do so.
    This solution is likely to be the model solution, simply because it makes your life so much easier when discussing novelty and IS. D2 will not be novel any longer because of being unsuitable for compositing with the light-sensitive earthworms, D1 has not water spraying device and D3 in addition thereto, also no lid. Clearly, D1 would now also be CPA and I think an IS attack based on D2 was not expected and could have dispensed with. If you would still do so, you might have had to argue that the light-sensitivity of worms/earthworms has not been perceived as a problem, i.e. you'll take the problem-invention approach. Improving the apparatus of D2 to also allow efficient composting by the worms (what earthworms in particular do automatically) would seem to use a pointer towards the solution when formulating the objective technical problem.
    Moreover, you satisfy most of the client's wishes without restricting the claim to a meaningless scope of protection, i.e. the water container issue.
    The key point was to use the client's indication of amending the preamble and do the opposite of what he suggested, i.e. further purpose-limiting the claim in the preamble. But honestly, what a bad way of hinting... candidates will start thinking about unallowed broadening and completely miss the main point. And if you missed it, and most of us did, there were no safeguards to come up with a good alternative claim 1 (most amendments would introduce severe limitations).
    Personally I also regret that if narrowing the preamble was the key point, then paper B has been drafted without too much care. I consider it a fundamental deficiency that the purpose of the transparent lid, the external light source, and their interrelation in doc D2 have not been set out clearly. The model solution is flawed because the container of D2 could still be considered as being suitable for composting organic refuse by earthworms since it contains the same features as the ideal claim 1 and the external light source does not necessarily have to be switched on at all times. If that is what you decided, then the relevance of D2 in the subsequent analysis is completely different and your life became much harder. A clear statement on the external light source was definitely missing.

    ReplyDelete
  67. continuing above comment

    The spraying of water on the refuse to add moisture during hot and dry days has been the underlying inventive concept justifying unity of the independent claims. However, you might actually wonder whether a CII in claim 6, which merely recommends addition of moisture, really is achieving this effect. Here, a more active step might have been necessary, but then you leave the realm of pure CII. You could of course include a step in which you provide a container of the previous claims, but then why still limiting to a CII; providing the container removes the exception from patentability objection. Anybody having suggestions on this topic?
    Overall a very tricky business, as even experienced people/professionals did not get it right in the little time available for the paper.

    ReplyDelete

Post a Comment

1 – 200 of 421 comments Newer Newest